Weak Closure, clarification












2












$begingroup$


I'm working on the following question:




Show that the closed unit ball $B(0,1)$ in a normed space is also weakly closed.




I think I want to show that $$lim_{nrightarrow infty}f(x_n) in f(B(0,1)) subset mathbb{R}$$ for all $f in X^*$



This is strange to me though because for each $f$, I will get a different image in $mathbb{R}$. This correct though right? I'm checking that the limit lives in the image, so long as the image is created by an element of $X^*$.



EDIT:
This question is not a duplicate of the linked question. In fact, I have an answer already. I wanted to get input on this idea that "I'm checking if the limit lives in ALL images formed from elements of $X^*$" is a way to think about this problem because I've never heard it posed that way before.










share|cite|improve this question











$endgroup$








  • 2




    $begingroup$
    Possible duplicate of closed unit ball in a Banach space is closed in the weak topology
    $endgroup$
    – aduh
    Dec 4 '18 at 15:50










  • $begingroup$
    You cannot show weak closedness by checking closedness against weakly converging sequences.
    $endgroup$
    – daw
    Dec 5 '18 at 7:22
















2












$begingroup$


I'm working on the following question:




Show that the closed unit ball $B(0,1)$ in a normed space is also weakly closed.




I think I want to show that $$lim_{nrightarrow infty}f(x_n) in f(B(0,1)) subset mathbb{R}$$ for all $f in X^*$



This is strange to me though because for each $f$, I will get a different image in $mathbb{R}$. This correct though right? I'm checking that the limit lives in the image, so long as the image is created by an element of $X^*$.



EDIT:
This question is not a duplicate of the linked question. In fact, I have an answer already. I wanted to get input on this idea that "I'm checking if the limit lives in ALL images formed from elements of $X^*$" is a way to think about this problem because I've never heard it posed that way before.










share|cite|improve this question











$endgroup$








  • 2




    $begingroup$
    Possible duplicate of closed unit ball in a Banach space is closed in the weak topology
    $endgroup$
    – aduh
    Dec 4 '18 at 15:50










  • $begingroup$
    You cannot show weak closedness by checking closedness against weakly converging sequences.
    $endgroup$
    – daw
    Dec 5 '18 at 7:22














2












2








2





$begingroup$


I'm working on the following question:




Show that the closed unit ball $B(0,1)$ in a normed space is also weakly closed.




I think I want to show that $$lim_{nrightarrow infty}f(x_n) in f(B(0,1)) subset mathbb{R}$$ for all $f in X^*$



This is strange to me though because for each $f$, I will get a different image in $mathbb{R}$. This correct though right? I'm checking that the limit lives in the image, so long as the image is created by an element of $X^*$.



EDIT:
This question is not a duplicate of the linked question. In fact, I have an answer already. I wanted to get input on this idea that "I'm checking if the limit lives in ALL images formed from elements of $X^*$" is a way to think about this problem because I've never heard it posed that way before.










share|cite|improve this question











$endgroup$




I'm working on the following question:




Show that the closed unit ball $B(0,1)$ in a normed space is also weakly closed.




I think I want to show that $$lim_{nrightarrow infty}f(x_n) in f(B(0,1)) subset mathbb{R}$$ for all $f in X^*$



This is strange to me though because for each $f$, I will get a different image in $mathbb{R}$. This correct though right? I'm checking that the limit lives in the image, so long as the image is created by an element of $X^*$.



EDIT:
This question is not a duplicate of the linked question. In fact, I have an answer already. I wanted to get input on this idea that "I'm checking if the limit lives in ALL images formed from elements of $X^*$" is a way to think about this problem because I've never heard it posed that way before.







functional-analysis soft-question






share|cite|improve this question















share|cite|improve this question













share|cite|improve this question




share|cite|improve this question








edited Dec 4 '18 at 15:53







yoshi

















asked Dec 4 '18 at 15:43









yoshiyoshi

1,186817




1,186817








  • 2




    $begingroup$
    Possible duplicate of closed unit ball in a Banach space is closed in the weak topology
    $endgroup$
    – aduh
    Dec 4 '18 at 15:50










  • $begingroup$
    You cannot show weak closedness by checking closedness against weakly converging sequences.
    $endgroup$
    – daw
    Dec 5 '18 at 7:22














  • 2




    $begingroup$
    Possible duplicate of closed unit ball in a Banach space is closed in the weak topology
    $endgroup$
    – aduh
    Dec 4 '18 at 15:50










  • $begingroup$
    You cannot show weak closedness by checking closedness against weakly converging sequences.
    $endgroup$
    – daw
    Dec 5 '18 at 7:22








2




2




$begingroup$
Possible duplicate of closed unit ball in a Banach space is closed in the weak topology
$endgroup$
– aduh
Dec 4 '18 at 15:50




$begingroup$
Possible duplicate of closed unit ball in a Banach space is closed in the weak topology
$endgroup$
– aduh
Dec 4 '18 at 15:50












$begingroup$
You cannot show weak closedness by checking closedness against weakly converging sequences.
$endgroup$
– daw
Dec 5 '18 at 7:22




$begingroup$
You cannot show weak closedness by checking closedness against weakly converging sequences.
$endgroup$
– daw
Dec 5 '18 at 7:22










1 Answer
1






active

oldest

votes


















1












$begingroup$

First, you cannot show weak closedness by weakly converging sequences.



Second, (sequential) closedness usually means that limits of converging sequences of elements from the closed set are in the closed set as well. For weak convergence, this has nothing to do with any functionals. So you idea does not work.






share|cite|improve this answer









$endgroup$













    Your Answer





    StackExchange.ifUsing("editor", function () {
    return StackExchange.using("mathjaxEditing", function () {
    StackExchange.MarkdownEditor.creationCallbacks.add(function (editor, postfix) {
    StackExchange.mathjaxEditing.prepareWmdForMathJax(editor, postfix, [["$", "$"], ["\\(","\\)"]]);
    });
    });
    }, "mathjax-editing");

    StackExchange.ready(function() {
    var channelOptions = {
    tags: "".split(" "),
    id: "69"
    };
    initTagRenderer("".split(" "), "".split(" "), channelOptions);

    StackExchange.using("externalEditor", function() {
    // Have to fire editor after snippets, if snippets enabled
    if (StackExchange.settings.snippets.snippetsEnabled) {
    StackExchange.using("snippets", function() {
    createEditor();
    });
    }
    else {
    createEditor();
    }
    });

    function createEditor() {
    StackExchange.prepareEditor({
    heartbeatType: 'answer',
    autoActivateHeartbeat: false,
    convertImagesToLinks: true,
    noModals: true,
    showLowRepImageUploadWarning: true,
    reputationToPostImages: 10,
    bindNavPrevention: true,
    postfix: "",
    imageUploader: {
    brandingHtml: "Powered by u003ca class="icon-imgur-white" href="https://imgur.com/"u003eu003c/au003e",
    contentPolicyHtml: "User contributions licensed under u003ca href="https://creativecommons.org/licenses/by-sa/3.0/"u003ecc by-sa 3.0 with attribution requiredu003c/au003e u003ca href="https://stackoverflow.com/legal/content-policy"u003e(content policy)u003c/au003e",
    allowUrls: true
    },
    noCode: true, onDemand: true,
    discardSelector: ".discard-answer"
    ,immediatelyShowMarkdownHelp:true
    });


    }
    });














    draft saved

    draft discarded


















    StackExchange.ready(
    function () {
    StackExchange.openid.initPostLogin('.new-post-login', 'https%3a%2f%2fmath.stackexchange.com%2fquestions%2f3025724%2fweak-closure-clarification%23new-answer', 'question_page');
    }
    );

    Post as a guest















    Required, but never shown

























    1 Answer
    1






    active

    oldest

    votes








    1 Answer
    1






    active

    oldest

    votes









    active

    oldest

    votes






    active

    oldest

    votes









    1












    $begingroup$

    First, you cannot show weak closedness by weakly converging sequences.



    Second, (sequential) closedness usually means that limits of converging sequences of elements from the closed set are in the closed set as well. For weak convergence, this has nothing to do with any functionals. So you idea does not work.






    share|cite|improve this answer









    $endgroup$


















      1












      $begingroup$

      First, you cannot show weak closedness by weakly converging sequences.



      Second, (sequential) closedness usually means that limits of converging sequences of elements from the closed set are in the closed set as well. For weak convergence, this has nothing to do with any functionals. So you idea does not work.






      share|cite|improve this answer









      $endgroup$
















        1












        1








        1





        $begingroup$

        First, you cannot show weak closedness by weakly converging sequences.



        Second, (sequential) closedness usually means that limits of converging sequences of elements from the closed set are in the closed set as well. For weak convergence, this has nothing to do with any functionals. So you idea does not work.






        share|cite|improve this answer









        $endgroup$



        First, you cannot show weak closedness by weakly converging sequences.



        Second, (sequential) closedness usually means that limits of converging sequences of elements from the closed set are in the closed set as well. For weak convergence, this has nothing to do with any functionals. So you idea does not work.







        share|cite|improve this answer












        share|cite|improve this answer



        share|cite|improve this answer










        answered Dec 5 '18 at 7:24









        dawdaw

        24.1k1544




        24.1k1544






























            draft saved

            draft discarded




















































            Thanks for contributing an answer to Mathematics Stack Exchange!


            • Please be sure to answer the question. Provide details and share your research!

            But avoid



            • Asking for help, clarification, or responding to other answers.

            • Making statements based on opinion; back them up with references or personal experience.


            Use MathJax to format equations. MathJax reference.


            To learn more, see our tips on writing great answers.




            draft saved


            draft discarded














            StackExchange.ready(
            function () {
            StackExchange.openid.initPostLogin('.new-post-login', 'https%3a%2f%2fmath.stackexchange.com%2fquestions%2f3025724%2fweak-closure-clarification%23new-answer', 'question_page');
            }
            );

            Post as a guest















            Required, but never shown





















































            Required, but never shown














            Required, but never shown












            Required, but never shown







            Required, but never shown

































            Required, but never shown














            Required, but never shown












            Required, but never shown







            Required, but never shown







            Popular posts from this blog

            Le Mesnil-Réaume

            Ida-Boy-Ed-Garten

            web3.py web3.isConnected() returns false always